Menu Close

Author: Tinku Tara

484-27-

Question Number 62054 by hhghg last updated on 14/Jun/19 $$\mathrm{484}×\mathrm{27} \\ $$ Commented by Tony Lin last updated on 15/Jun/19 $$=\mathrm{11}^{\mathrm{2}} ×\mathrm{6}^{\mathrm{2}} ×\mathrm{3} \\ $$$$=\left(\mathrm{108}×\mathrm{11}\right)×\mathrm{11}…

Question-127589

Question Number 127589 by bramlexs22 last updated on 31/Dec/20 Answered by liberty last updated on 31/Dec/20 $$\left(\bullet\right)\:\mathrm{let}\:\mathrm{y}'=\mathrm{z}\:\Rightarrow\:\mathrm{xz}'\:+\:\mathrm{z}\:=\:\mathrm{3x}^{\mathrm{2}} −\mathrm{x}\: \\ $$$$\:\:\:\:\frac{\mathrm{d}}{\mathrm{dx}}\:\left(\mathrm{xz}\right)\:=\:\mathrm{3x}^{\mathrm{2}} −\mathrm{x}\: \\ $$$$\:\:\:\mathrm{xz}\:=\:\mathrm{x}^{\mathrm{3}} −\frac{\mathrm{1}}{\mathrm{2}}\mathrm{x}^{\mathrm{2}} \:+\mathrm{C}_{\mathrm{1}}…

3-1-4-3-4-

Question Number 62052 by hhghg last updated on 14/Jun/19 $$\mathrm{3}\frac{\mathrm{1}}{\mathrm{4}}−\frac{\mathrm{3}}{\mathrm{4}} \\ $$ Commented by maxmathsup by imad last updated on 15/Jun/19 $$=\frac{\mathrm{3}}{\mathrm{4}}−\frac{\mathrm{3}}{\mathrm{4}}=\mathrm{0} \\ $$ Terms…

3-5-3-3-3-2-

Question Number 62050 by hhghg last updated on 14/Jun/19 $$\left[\mathrm{3}×\mathrm{5}+\mathrm{3}\right]+\left[\mathrm{3}+\left(\mathrm{3}×\mathrm{2}\right)\right] \\ $$ Commented by meme last updated on 15/Jun/19 $$=\mathrm{18}+\mathrm{9} \\ $$$$=\mathrm{27} \\ $$ Terms…

1-4-1-5-

Question Number 62049 by hhghg last updated on 14/Jun/19 $$\frac{\mathrm{1}}{\mathrm{4}}+\frac{\mathrm{1}}{\mathrm{5}} \\ $$ Commented by maxmathsup by imad last updated on 15/Jun/19 $$=\frac{\left(\mathrm{1}×\mathrm{5}\right)+\left(\mathrm{4}×\mathrm{1}\right)}{\mathrm{4}×\mathrm{5}}\:=\frac{\mathrm{5}+\mathrm{4}}{\mathrm{20}}\:=\frac{\mathrm{9}}{\mathrm{20}} \\ $$ Terms…

1-2-3-

Question Number 62048 by hhghg last updated on 14/Jun/19 $$\left(\frac{\mathrm{1}}{\mathrm{2}}\right)^{\mathrm{3}} \\ $$ Commented by maxmathsup by imad last updated on 15/Jun/19 $$=\frac{\mathrm{1}^{\mathrm{3}} }{\mathrm{2}^{\mathrm{3}} }\:=\frac{\mathrm{1}}{\mathrm{8}} \\…